Immune- System Activity

This topic has expert replies
Master | Next Rank: 500 Posts
Posts: 154
Joined: Thu Mar 19, 2009 12:55 pm
Thanked: 5 times
Followed by:1 members

Immune- System Activity

by vinaynp » Sat May 16, 2009 8:27 pm
A researcher discovered that people who have low levels of immune-system activity tend to score much lower on tests of mental health than do people with normal or high immune-system activity. The researcher concluded that the immune system protects against mental illness as well as against physical disease.


The researcher's conclusion depends on which of the following assumptions?

a) High immune-stem activity protects against mental illness better than normal immune-system activity does.
b) Mental illness is similar to physical disease in its effects on body systems.
c) People with high immune-system activity cannot develop mental illness.
d) Mental illness does not cause people's immune-system activity to decrease.
e) Psychological treatment of mental illness is not as effective as is medical treatment.

Please answer only with explanations.

Master | Next Rank: 500 Posts
Posts: 252
Joined: Mon Feb 02, 2009 6:34 am
Thanked: 305 times
Followed by:55 members
GMAT Score:760

by myohmy » Sat May 16, 2009 9:52 pm
I would go with D in this case.


a) High immune-stem activity protects against mental illness better than normal immune-system activity does.

We can get rid of this one since it talks about high immune stem activity versus normal immune stem activity, while the question addresses high/normal versus low.


b) Mental illness is similar to physical disease in its effects on body systems.

The conclusion doesn't indicate that mental illness has similar effects on bodily systems, only that they are less prevalent on people with low immune stem activity.


c) People with high immune-system activity cannot develop mental illness.

This choice is too extreme to be correct.


d) Mental illness does not cause people's immune-system activity to decrease.

This is the necessary assumption. If mental illness causes someone's immune-system activity to decrease, then people with previously normal/high immune system activity would decrease to low activity when they became mentally ill. Thus, we could not conclude that the immune system prevented mental illness. Therefore, we must assume that mental illness has no effect on immune system activity.


e) Psychological treatment of mental illness is not as effective as is medical treatment.

This is irrelevant to the conclusion.


IMO D is the correct answer.

User avatar
Legendary Member
Posts: 682
Joined: Fri Jan 16, 2009 2:40 am
Thanked: 32 times
Followed by:1 members

Re: Immune- System Activity

by Vemuri » Sun May 17, 2009 3:16 am
Premise: A researcher discovered that people with low level of immune-system score much lower on tests of mental health than do people with normal or high immune-system activity.

Conclusion: The researcher concluded that the immune system protects against mental illness as well as against physical disease.

A - The difference between High & normal immune-system activity is irrelavent.
B - The effects of mental illness & physical disease on body systems is not relavant.
C - People with high immune-system cannot develop mental illness. If we use the negation test on this one, it will be "People with high immune-system can develop mental illness". Will this weaken the conclusion? Yes, it does because the conclusion of the researcher states that immune system protects against mental illness as well as physical disease". Keep this option aside.
D - Mental illness does not cause people's immune-system activity to decrease. Use the negation test on this one, it will be "Mental illness does cause people's immune-system activity to decrease". Does this weaken the conclusion? Does not do anything.
E - Irrelavant

IMO C

Master | Next Rank: 500 Posts
Posts: 154
Joined: Thu Mar 19, 2009 12:55 pm
Thanked: 5 times
Followed by:1 members

by vinaynp » Sun May 17, 2009 9:36 am
OA is D . It is an OG problem given in 300 real questions.

User avatar
Master | Next Rank: 500 Posts
Posts: 229
Joined: Tue Jan 13, 2009 6:56 am
Thanked: 8 times
GMAT Score:700

by Uri » Sun May 17, 2009 2:34 pm
the answer has to be (D). it is a defender type of assumption.

Master | Next Rank: 500 Posts
Posts: 154
Joined: Thu Mar 19, 2009 12:55 pm
Thanked: 5 times
Followed by:1 members

by vinaynp » Sun May 17, 2009 6:02 pm
Can you throw some more light on defender type of questions?

User avatar
Master | Next Rank: 500 Posts
Posts: 385
Joined: Sun May 24, 2009 3:55 pm
Thanked: 11 times
GMAT Score:740

by Domnu » Sun May 24, 2009 5:24 pm
Hmm.. well I would go with choice D mainly because it is the contrapositive of a statement provided in the passage.
Have you wondered how you could have found such a treasure? -T

Junior | Next Rank: 30 Posts
Posts: 26
Joined: Thu Dec 11, 2008 12:02 am
Thanked: 1 times

by francopiccolo » Mon May 25, 2009 2:19 pm
The researcher concludes that a low inmmune system activity is the cause of mental illness, but it could be the other way round. Hence D.

Legendary Member
Posts: 727
Joined: Sun Jun 08, 2008 9:32 pm
Thanked: 8 times
Followed by:1 members

by umaa » Sun Aug 23, 2009 12:14 pm
I couldn't find out why B is wrong.

The conclusion says about both mental illness and physical decease. But the premise talks only about mental illness. So, the assumption should be both about mental illness and physical decease.

Am I right? May I know why B is wrong.. I'm not convinced with OG explanations.
What we think, we become

Senior | Next Rank: 100 Posts
Posts: 52
Joined: Tue Aug 05, 2014 3:20 am

by Eric77Gorm » Mon May 16, 2016 12:07 am
In my opinion D is the most logical one.